You are on page 1of 62

2/3 Question 1-3 of 71

Theme: Amputations

A. Transfemoral amputation
B. Gritti - Stokes amputation
C. Digital amputation
D. Syme's amputation
E. Hindquarter amputation
F. Below knee amputation
G. Trans metatarsal amputation
H. Amputation of digit

Please select the most appropriate procedure for the scenario given. Each option may be used once, more than
once or not at all.

1. The operation of choice for a 90 year old lady with infected gangrene of the mid foot secondary to
diabetes. She has fixed flexion deformity of the knee.

You answered Syme's amputation

The correct answer is Transfemoral amputation

An elderly patient with diabetes and peripheral vascular disease is a high risk surgical candidate. It is
important that the chances of a successful outcome are maximised at the first operation. SInce above
knee amputations usually heal more reliably than below knee amputations this is a preferable option,
especially since she has a fixed deformity.

2. An operation in which Skew flaps are created.

Below knee amputation

This is one variant of a below knee amputation. The Burgess flap is the other commonly practised
approach.

3. An amputation of the lower limb in which the femoral condyles are removed and the patella retained.

Gritti - Stokes amputation

This is a Gritti - Stokes amputation. During a Gritti - Stokes operation the patella is conserved and
swung posteriorly to cover the distal femoral surface.
Beware performing amputations in patients with peripheral vascular disease without optimising inflow first!

Amputations
Amputations are indicated when the affected limb is one of the following:

• Dead non viable


• Deadly where it is posing a major threat to life
• Dead useless where it is viable but a prosthesis would be preferable

Orthopaedic surgery

• Amputation is often undertaken as an option of last resort e.g. Limb salvage has failed and the limb is so
non functional that mobility needs would be best met with prosthesis.
• Chronic fracture non union or significant limb shortening following trauma would fit into this category.
Occasionally following major trauma a primary amputation is preferable. This would be the case in an
open fracture with major distal neurovascular compromise and other more life threatening injuries are
present.

Vascular surgery

• The first two categories are the most prevalent.


• Diabetic foot sepsis is often a major cause of sepsis which can spread rapidly in the presence of
established peripheral vascular disease.
• As a general rule the main issue in vascular surgery is to optimise vascular inflow prior to surgery. The
more distal the planned amputation is to be, the more important this rule becomes.
• In other situations there has been something such as an embolic event that has not been revascularised in
time. In this case the limb shows fixed mottling and an amputation will be needed.

Types of amputations
As the vast majority of commonly performed amputations affect the lower limbs these will be covered here.

The main categories of amputations are:

• Pelvic disarticulation (hindquarter)


• Above knee amputation
• Gritti Stokes (through knee amputation)
• Below knee amputation (using either Skew or Burgess flaps)
• Syme's amputation (through ankle)
• Amputations of mid foot and digits

Choosing a level of amputation depends on:

• The disease process being treated


• Desired functional outcome
• Co-morbidities of the patient

Above knee amputations


• Quick to perform
• Heal reliably
• Patients regain their general health quickly
• For this benefit, a functional price has to be paid and many patients over the age of 70 will never walk
on an above knee prosthesis.
• Above knee amputations use equal anterior-posterior flaps

Below knee amputations

• Technically more challenging to perform


• Heal less reliably than their above knee counterparts.
• However, many more patients are able to walk using a below knee prosthesis.
• In below knee amputations the two main flaps are Skew flaps or the Burgess

Long posterior flap. There is some evidence that Skew flaps are better
vascularised than the long posterior flap and some vascular surgeons prefer
them for this reason.

It is worth remembering that whilst it may be technically feasible to offer a below knee amputation there may
be circumstances where an above knee option is preferable. For example, in fixed flexion deformities of the
lower limb, little functional benefit would be gained from below knee amputation surgery.

1/3 Question 4-6 of 71


Theme: Acute limb ischaemia

A. Primary amputation
B. Transfemoral embolectomy with prophylactic fasciotomy
C. Transpopliteal embolectomy without prophylactic fasciotomy
D. Transfemoral embolectomy without prophylactic fasciotomy
E. Transpopliteal embolectomy with prophylactic fasciotomy
F. Angiogram
G. Systemic heparin infusion
H. Peripheral thrombolysis

Please select the most appropriate management for the following patients presenting with acutely ischaemic
limbs. Each option may be used once, more than once or not at all.

4. A 76 year old man presents with a painful left leg. The pain began suddenly and with no previous
history. On examination he has a white left leg with no palpable femoral pulse and loss of sensation.
The pulses in the contra lateral limb are normal. It is now three hours since the pain first started.

You answered Transfemoral embolectomy with prophylactic fasciotomy

The correct answer is Transfemoral embolectomy without prophylactic fasciotomy


A limb which is acutely ischaemic and with normal contralateral pulses normally indicates an acute
embolus. Whilst intra arterial thrombolysis may be an option there is a reasonable argument for
immediate surgery. A fasciotomy is unlikely to be required.

5. A 56 year old man presents with a painful left leg. The pain has been present for the past 8 hours
although it has also been present (though less severe) about a week ago. At that stage he noted that his
hallux had turned blue. This resolved spontaneously. On examination he has a weakly palpable
femoral pulse on the affected side but no pulses palpable distal to this. His sensation is mildly
impaired.

Angiogram

The history favors a more chronic process and the great toe cyanotic spell may be indicative of
previous embolism from pathology such as an aneurysm. In the ideal scenario a duplex scan would be
performed. However, an angiogram would probably supply sufficient information and allow
appropriate endovascular therapy.

6. A 78 year old lady is found by carers with a severely painful left leg. On examination she has no
palpable pulses and the limb is cold, insensate and mottled. The mottling does not blanch with
pressure.

You answered Peripheral thrombolysis

The correct answer is Primary amputation

This is an unsalvagable limb and is best amputated primarily.

Acute limb ischaemia

• Thrombosis of a pre-existing site of atherosclerosis if the commonest cause of acute limb ischamia
• Acute thrombosis of popliteal anuerysms poses the greatest threat to the limb
• Sudden occlusion of a large proximal vessel results in the typical appearances of acute limb ischaemia

Clinical appearances

• Less than 6 hours = White leg


• At 6-12 hours mottled limb with blanching on pressure
• More than 12-24 hours - fixed mottling

Management of acutely ischaemic leg


Clinical picture Treatment
White leg with sensorimotor deficit Surgery and embolectomy
Dusky leg, mild anaestheia Angiography
Fixed mottling Primary amputation

Role of thrombolysis

• Intra arterial thrombolysis is better than peripheral thrombolysis


• Mainly inidicated in acute on chronic thrombosis
• Avoid if within 2 months of CVA or 2 weeks of surgery
• Aspiration of clot may improve success rate if the thrombosis is large

Surgery

• Both groins should be prepared


• Transverse arteriotomy is easier to close
• Poor inflow should be managed with iliac trawl- if this fails to improve then consider a femoro-femoral
cross over or axillo-femoral cross over.
• A check angiogram should be performed on table and prior to closure
• Systemic heparinisation should follow surgery
• Fasciotomy should be considered if the time between onset and surgery exceeds 6 hours

2/3 Question 7-9 of 71


Theme: Vascular disorders affecting the upper limb

A. Proximal brachial artery occlusion secondary to atheroma


B. Distal brachial artery occlusion secondary to atheroma
C. Axillary artery embolus
D. Axillary vein thrombosis
E. Cervical rib
F. Raynaud's disease
G. Rheumatoid disease

Please select the most likely cause for the presenting scenario described. Each option may be used once, more
than once or not at all.

7. A 73 year old male presents with a collapse and is brought to the emergency department. On
examination he has a cold, painful left hand and forearm.

You answered Proximal brachial artery occlusion secondary to atheroma

The correct answer is Axillary artery embolus

Theme from September 2012 Exam


Sudden arterial embolus will affect the axillary artery in up to 30% cases. Because of the acute nature
of the condition there is no time for the development of a collateral circulation so the limb is usually
pale and painful. Emboli occur usually occur as a result of atrial fibrillation. Fast atrial fibrillation can
cause syncope and an acute embolus.
8. A 23 year old man presents with intermittent symptoms of altered sensation in his arm and discomfort
when he uses his hands. He works as an electrician and his symptoms are worst when he is fitting light
fixtures.

Cervical rib

Compression of the thoracic outlet by the fibrous band of the "rib" can result in both neurological and
circulatory compromise. When manual tasks are performed in which the hand works overhead the
signs and symptoms will be maximal and this is the basis of Adsons test.

9. A 19 year old lady presents with recurrent episodes of pain in her hands. She notices that her
symptoms are worst in cold weather. When she gets the pain she notices that her hands are very pale,
they then become dark blue in colour.

Raynaud's disease

Raynauds disease is characterised by a series of colour changes and discomfort is often present. The
young age at presentation coupled with the absence of a smoking history (in most cases) makes
occlusive disease unlikely.

Vascular disorders of the upper limb

Upper limb arterial disease is less common than lesions causing symptoms in the lower limb. The upper limb
circulation may be affected by embolic events, stenotic lesions (both internal and extrinsic), inflammatory
disorders and venous diseases.
The anatomy of the collateral circulation of the arterial inflow may impact on the history and nature of disease
presentation. In the region of the subclavian and axillary arteries the collateral vessels passing around the
shoulder joint may provide pathways for flow if the main vessels are stenotic or occluded. During periods of
increased metabolic demand the collateral flow is not sufficient and the vertebral arteries may have diminished
flow. This may result in diminished flow to the brain with neurological sequelae such as syncope.

Vascular disease of the upper limb


Condition Features
Axillary/ • 50% of upper limb emboli will lodge in the brachial artery
brachial • 30% of upper limb emboli will lodge in the axillary artery
embolus • Sudden onset of symptoms; pain, pallor, paresis, pulselessness, paraesthesia
• Sources are left atrium with cardiac arrhythmia (mainly AF), mural thrombus
• Cardiac arrhythmias may cause result in impaired consciousness in addition to the
embolus

Arterial • Those resulting from atheroma are the most common, trauma may result in vascular
occlusions changes and long term occlusion but this is rare
• Features may include claudication, ulceration and gangrene. Proximally sited lesions
may result in subclavian steal syndrome
• The progressive nature of the disease allows development of collaterals, acute
ischaemia may occur as a result of acute thrombosis

Raynaud's • Idiopathic condition affecting young females


disease • Usually affects hands > feet
• Digits become: white -->blue -->red
• Treatment is with calcium antagonists

Upper limb • Gradual onset of upper limb swelling and discomfort.


venous • Sensation and motor function are normal
thrombosis • Condition may complicate pre-existing malignancy (especially breast cancer) or arise
as a result of repetitive use of the limb in a task such as painting a ceiling
• The condition is diagnosed with duplex ultrasound and treatment is with
anticoagulation

Cervical rib • 0.2-0.4% incidence


• Consist of an anomalous fibrous band that often originates from C7 and may arc
towards, but rarely reaches the sternum
• Congenital cases may present around the third decade, some cases are reported to
occur following trauma
• Bilateral in up to 70%
• Compression of the subclavian artery may produce absent radial pulse on clinical
examination and in particular may result in a positive Adsons test (lateral flexion of
the neck away from symptomatic side and traction of the symptomatic arm- leads to
obliteration of radial pulse)
• Treatment is most commonly undertaken when there is evidence of neurovascular
compromise. A transaxillary approach is the traditional operative method for excision

3/3 Question 10-12 of 71


Theme: Management of occlusive vascular disease

A. Aorto-bifemoral bypass graft


B. Femoro-femoral cross over graft
C. Femoro-popliteal bypass graft
D. Femoro-distal bypass graft
E. Axillo-bifemoral bypass graft
F. Bilateral above knee amputation

Please select the most appropriate arterial bypass method for the scenario described. Each option may be used
once, more than once or not at all.

10. An 83 year old lady with a significant cardiac history is admitted with rest pain and bilateral leg
ulcers. Imaging demonstrates bilateral occlusion of both common iliac arteries that are unsuitable for
stenting.

Axillo-bifemoral bypass graft

Theme from January 2012 Exam


In patients with major cardiac co-morbidities the safest option is to choose an axillo-bifemoral bypass
graft. The long term patency rates are less good than with aorto-bifemoral bypass grafts, however, the
operation is less major.

11. A 54 year old man presents to the vascular clinic with severe rest pain and an ulcer on his right foot
that is not healing. On examination he has bilateral absent femoral pulses. Imaging demonstrates a
bilateral occlusion of the common iliac arteries that is not suitable for stenting.

Aorto-bifemoral bypass graft

In a young patient consideration should be given to aorto-bifemoral bypass grafts as these have the
best long term functional outcome compared with an axillobifemoral bypass graft.

12. A 78 year old man presents with left sided rest pain in his leg and a non healing arterial leg ulcer on
the same leg. Imaging shows normal right leg vessels, on the left side there is a long occlusion of the
external iliac artery that is unsuitable for stenting. He has a significant cardiac history.

Femoro-femoral cross over graft

Femoro-femoral cross over grafts are an option for treatment of iliac occlusions in patients with
significant co-morbidities and healthy contralateral vessels. In reality the idealised situation presented
here seldom applies and the opposite vessels usually have some disease and one must be careful not
to damage the "healthy" side.

Peripheral vascular disease

Indications for surgery to revascularise the lower limb

• Intermittent claudication
• Critical ischaemia
• Ulceration
• Gangrene

Intermittent claudication that is not disabling may provide a relative indication, whilst the other complaints are
often absolute indications depending upon the frailty of the patient.

Assessment

• Clinical examination
• Ankle brachial pressure index measurement
• Duplex arterial ultrasound
• Angiography (standard, CT or MRI): usually performed only if intervention being considered.
Angioplasty
In order for angioplasty to be undertaken successfully the artery has to be accessible. The lesion relatively short
and reasonable distal vessel runoff. Longer lesions may be amenable to sub-intimal angioplasty.

Surgery
Surgery will be undertaken where attempts at angioplasty have either failed or are unsuitable. Bypass essentially
involves bypassing the affected arterial segment by utilising a graft to run from above the disease to below the
disease. As with angioplasty good runoff improves the outcome.

Some key concepts with bypass surgery

Superficial femoral artery occlusion to the above knee

• Angioplasty may be attempted but otherwise these patients will require a femoro-popliteal bypass graft.
• Patency rates for Polytetrafluoroethylene (PTFE) and vein are similar, so PTFE preferred unless co-
existing infection makes use of prosthetic material undesirable.

Procedure

• Artery dissected out, IV heparin 3,000 units given and then the vessels are cross clamped
• Longitudinal arteriotomy
• Graft cut to size and tunneled to arteriotomy sites
• Anastomosis to femoral artery usually with 5/0 'double ended' Prolene suture
• Distal anastomosis usually using 6/0 'double ended' Prolene

Distal disease

• Femoro-distal bypass surgery takes longer to perform, is more technically challenging and has higher
failure rates.
• In elderly diabetic patients with poor runoff a primary amputation may well be a safer and more
effective option. There is no point in embarking on this type of surgery in patients who are wheelchair
bound.
• In femorodistal bypasses vein gives superior outcomes to PTFE.

Rules

• Vein mapping 1st to see whether there is suitable vein (the preferred conduit). Sub intimal hyperplasia
occurs early when PTFE is used for the distal anastomosis and will lead to early graft occlusion and
failure.
• Essential operative procedure as for above knee fem-pop.
• If there is insufficient vein for the entire conduit then vein can be attached to the end of the PTFE graft
and then used for the distal anastomosis. This type of 'vein boot' is technically referred to as a Miller
Cuff and is associated with better patency rates than PTFE alone.
• Remember the more distal the arterial anastomosis the lower the success rate.

2/3 Question 13-15 of 71


Theme: Ankle brachial pressure index
A. >1.2
B. 1.0
C. 0.8
D. 0.5
E. 0.3

Please select the ankle brachial pressure index that is most likely to be present for the scenario given. Each
value may be used once, more than once or not at all.

13. A 73 year old lifelong heavy smoker presents to the vascular clinic with symptoms of foot ulceration
and rest pain. On examination her foot has areas of gangrene and pulses are impalpable.

0.3

This is critical limb ischaemia. Values of 0.3 are typical in this setting and urgent further imaging is
needed. Debridement of necrosis prior to improving arterial inflow carries a high risk of limb loss.

14. A 63 year old man presents with a claudication distance of 15 yards. He is a lifelong heavy smoker.
On examination his foot is hyperaemic and there is a small ulcer at the tip of his great toe.

0.5

Hyperaemia may occur in association with severe vascular disease and is referred to surgically as a
"sunset foot". ABPI is usually higher than 0.3, but seldom greater than 0.5. Especially when
associated with hyperaemic changes and ulceration. Urgent further imaging and risk factor
modification is needed.

15. A 77 year old morbidly obese man with type 2 diabetes presents with leg pain at rest. His symptoms
are worst at night and sometimes improve during the day. He has no areas of ulceration.

You answered 0.8

The correct answer is >1.2

Type 2 diabetes may have vessel calcification. This will result in abnormally high ABPI readings.
Pain of this nature in diabetics is usually neuropathic and if a duplex scan is normal then treatment
with an agent such as duloxetine is sometimes helpful.
Theme from September 2011 and September 2012 exam

Ankle-Brachial pressure index

• Measurement of ankle- brachial pressure index (ABPI) is a commonly performed vascular investigation.
• Calculated by dividing lower limb pressure by the highest upper limb pressure.
Results of ABPI
1.2 or greater Usually due to vessel calcification
1.0- 1.2 Normal
0.8-1.0 Minor stenotic lesion
Initiate risk factor management
0.50-0.8 Moderate stenotic lesion
Consider duplex
Risk factor management
If mixed ulcers present then avoid tight compression bandages
0.5- 0.3 Likely significant stenosis
Duplex scanning to delineate lesions needed
Compression bandaging contra indicated
Less than 0.3 Indicative of critical ischaemia
Urgent detailed imaging required

Question 16 of 71
A 67 year old patient is due to undergo a femoro-popliteal bypass graft. Which heparin regime should the
surgeon ask for prior to cross clamping the femoral artery?

A. Single therapeutic dose of low molecular weight heparin on the ward


prior to coming to theatre

B. Single therapeutic dose of low molecular weight heparin the night


before surgery

C. Dose of 10,000 units of unfractionated heparin prior to induction of


anaesthesia

D. Dose of 3,000 units of unfractionated heparin, 3 minutes prior to


cross clamping

E. Dose of 30,000 units of unfractionated heparin, 3 minutes prior to


cross clamping

As a rule most vascular surgeons will administer approximately 3,000 units of systemic heparin 3-5 minutes
prior to cross clamping to help prevent further intra arterial thromboses. A dose of 30,000 units is given prior to
going on cardiopulmonary bypass. Heparin given at induction will cause bleeding during routine dissection.

Heparin

• Causes the formation of complexes between antithrombin and activated thrombin/factors 7,9,10,11 & 12

Advantages of low molecular weight heparin

• Better bioavailability
• Lower risk of bleeding
• Longer half life
• Little effect on APTT at prophylactic dosages
• Less risk of HIT

Complications

• Bleeding
• Osteoporosis
• Heparin induced thrombocytopenia (HIT): occurs 5-14 days after 1st exposure
• Anaphylaxis

In surgical patients that may need a rapid return to theatre administration of unfractionated heparin is preferred
as low molecular weight heparins have a longer duration of action and are harder to reverse.

Question 17 of 71
An 18 year old lady presents with extensive varicose veins of her left leg. There is associated port wine staining.
What is the most likely diagnosis?

A. Type 1 diabetes

B. Osler syndrome

C. Gardner's syndrome

D. Proteus syndrome

E. Klippel-Trenaunay-Weber syndrome

A less common cause of venous insufficiency is Klippel-Trenaunay-Weber (KTW) syndrome, which involves
port-wine stains, varicose veins, and bony or soft-tissue hypertrophy.

Lower leg ulcers

Venous leg ulcers

• Most due to venous hypertension, secondary to chronic venous insufficiency (other causes include calf
pump dysfunction or neuromuscular disorders)
• Ulcers form due to capillary fibrin cuff or leucocyte sequestration
• Features of venous insufficiency include oedema, brown pigmentation, lipodermatosclerosis, eczema
• Location above the ankle, painless
• Deep venous insufficiency is related to previous DVT and superficial venous insufficiency is associated
with varicose veins
• Doppler ultrasound looks for presence of reflux and duplex ultrasound looks at the anatomy/ flow of the
vein
• Management: 4 layer compression banding after exclusion of arterial disease or surgery
• If fail to heal after 12 weeks or >10cm2 skin grafting may be needed

Marjolin's ulcer

Image sourced from Wikipedia

• Squamous cell carcinoma


• Occurring at sites of chronic inflammation e.g; burns, osteomyelitis after 10-20 years
• Mainly occur on the lower limb

Arterial ulcers

• Occur on the toes and heel


• Painful
• There may be areas of gangrene
• Cold with no palpable pulses
• Low ABPI measurements

Neuropathic ulcers

• Commonly over plantar surface of metatarsal head and plantar surface of hallux
• The plantar neuropathic ulcer is the condition that most commonly leads to amputation in diabetic
patients
• Due to pressure
• Management includes cushioned shoes to reduce callous formation

Pyoderma gangrenosum
Image sourced from Wikipedia

• Associated with inflammatory bowel disease/RA


• Can occur at stoma sites
• Erythematous nodules or pustules which ulcerate

3/3 Question 18-20 of 71


Theme: Management of peripheral arterial disease

A. Primary amputation
B. Angioplasty
C. Arterial bypass surgery using vein
D. Arterial bypass surgery using PTFE
E. Conservative management with medical therapy and exercise
F. Watch and wait
G. Duplex scanning

Please select the most appropriate management for the scenario given. Each option may be used once, more
than once or not at all.

18. A 63 year old man is admitted with rest pain and foot ulceration. An angiogram shows a 3 cm area of
occlusion of the distal superficial femoral artery with 3 vessel run off. His ankle - brachial pressure
index is 0.4.

Angioplasty

Short segment disease and good run off with tissue loss is a compelling indication for angioplasty. He
should receive aspirin and a statin if not already taking them.

19. A 72 year old man present in the vascular clinic with calf pain present on walking 100 yards. He is an
ex-smoker and lives alone. On examination he has reasonable leg pulses. His right dorsalis pedis
pulse gives a monophasic doppler signal with an ankle brachial pressure index measurement of 0.7.
All other pressures are acceptable. There is no evidence of ulceration or gangrene.

Conservative management with medical therapy and exercise

Structured exercise programmes combined with medical therapy will improve many patients. Should
his symptoms worsen or fail to improve then imaging with duplex scanning would be required.

20. An 83 year old lady is admitted from a nursing home with infected lower leg ulcers. She underwent
an attempted long superficial femoral artery sub initimal angioplasty 2 weeks previously. This
demonstrated poor runoff below the knee.

Primary amputation

Poor runoff and sepsis would equate to poor outcome with attempted bypass surgery.

Peripheral vascular disease

Indications for surgery to revascularise the lower limb

• Intermittent claudication
• Critical ischaemia
• Ulceration
• Gangrene

Intermittent claudication that is not disabling may provide a relative indication, whilst the other complaints are
often absolute indications depending upon the frailty of the patient.

Assessment

• Clinical examination
• Ankle brachial pressure index measurement
• Duplex arterial ultrasound
• Angiography (standard, CT or MRI): usually performed only if intervention being considered.

Angioplasty
In order for angioplasty to be undertaken successfully the artery has to be accessible. The lesion relatively short
and reasonable distal vessel runoff. Longer lesions may be amenable to sub-intimal angioplasty.

Surgery
Surgery will be undertaken where attempts at angioplasty have either failed or are unsuitable. Bypass essentially
involves bypassing the affected arterial segment by utilising a graft to run from above the disease to below the
disease. As with angioplasty good runoff improves the outcome.
Some key concepts with bypass surgery

Superficial femoral artery occlusion to the above knee

• Angioplasty may be attempted but otherwise these patients will require a femoro-popliteal bypass graft.
• Patency rates for Polytetrafluoroethylene (PTFE) and vein are similar, so PTFE preferred unless co-
existing infection makes use of prosthetic material undesirable.

Procedure

• Artery dissected out, IV heparin 3,000 units given and then the vessels are cross clamped
• Longitudinal arteriotomy
• Graft cut to size and tunneled to arteriotomy sites
• Anastomosis to femoral artery usually with 5/0 'double ended' Prolene suture
• Distal anastomosis usually using 6/0 'double ended' Prolene

Distal disease

• Femoro-distal bypass surgery takes longer to perform, is more technically challenging and has higher
failure rates.
• In elderly diabetic patients with poor runoff a primary amputation may well be a safer and more
effective option. There is no point in embarking on this type of surgery in patients who are wheelchair
bound.
• In femorodistal bypasses vein gives superior outcomes to PTFE.

Rules

• Vein mapping 1st to see whether there is suitable vein (the preferred conduit). Sub intimal hyperplasia
occurs early when PTFE is used for the distal anastomosis and will lead to early graft occlusion and
failure.
• Essential operative procedure as for above knee fem-pop.
• If there is insufficient vein for the entire conduit then vein can be attached to the end of the PTFE graft
and then used for the distal anastomosis. This type of 'vein boot' is technically referred to as a Miller
Cuff and is associated with better patency rates than PTFE alone.
• Remember the more distal the arterial anastomosis the lower the success rate.

Question 21 of 71
A 32 year old woman attends clinic for assessment of varicose veins. She has suffered for varicose veins for
many years and can trace their development back to when she suffered a complex tibial fracture. On
examination she has marked truncal varicosities with a long tortuous long saphenous vein. What is the most
appropriate next step?

A. Arrange a venogram

B. Arrange a venous duplex scan


C. List her for a trendelenberg procedure

D. List her for injection foam sclerotherapy

E. List her for multiple avulsion phlebectomies

This lady is likely to have deep venous incompetence as she will have been immobilised for her tibial fracture
and may well have had a DVT. A duplex scan is mandatory prior to any form of surgical intervention. A
venogram would provide similar information but is more invasive.

Chronic venous insufficiency and Varicose veins

Wide spectrum of disease ranging from minor cosmetic problem through to ulceration and disability. It is
commoner in women than men and is worse during pregnancy.

• Defined as saccular dilatation of veins (WHO)

The veins of the lower limb consist of an interconnected network of superficial and deep venous systems.
Varices occur because of localised weakness in the vein wall resulting in dilatation and reflux of blood due to
non union of valve cusps.

• Histology: fibrous scar tissue dividing smooth muscle within media in the vessel wall

Tissue damage in chronic venous insufficiency occurs because of perivascular cytokine leakage resulting in
localised tissue damage coupled with impaired lymphatic flow.

• Affected veins: normally long and short saphenous veins

Diagnosis
Typical symptoms of varicose veins include:

• Cosmetic appearance
• Aching
• Ankle swelling that worsens as the day progresses
• Episodic thrombophlebitis
• Bleeding
• Itching

Symptoms of chronic venous insufficiency include:

• Dependant leg pain


• Prominent leg swelling
• Oedema extending beyond the ankle
• Venous stasis ulcers
The typical venous stasis ulcer is:

• Located above the medial malleolus


• Indolent appearance with basal granulation tissue
• Variable degree of scarring
• Non ischaemic edges
• Haemosiderin deposition in the gaiter area (and also lipodermatosclerosis).

Differential diagnosis

• Lower limb arterial disease


• Marjolins ulcer
• Claudication
• Spinal stenosis
• Swelling due to medical causes e.g. CCF.

Exclusion of these differentials is by means of physical examination and ankle brachial pressure index
measurement.

Examination

• Assess for dilated short saphenous vein (popliteal fossa) and palpate for saphena varix medial to the
femoral artery
• Brodie-Trendelenburg test: to assess level of incompetence
• Perthes' walking test: assess if deep venous system competent

Investigation

• Doppler exam: if incompetent a biphasic signal due to retrograde flow is detected


• Duplex scanning: to ensure patent deep venous system (do if DVT or trauma)

All patients should have a Doppler assessment to assess for venous reflux and should be classified as having
uncomplicated varicose veins or varicose veins with associated chronic venous insufficiency. In the history
establishing a previous thrombotic event (DVT/ lower limb fracture) is important and patients with such a
history and all who have evidence of chronic venous insufficiency should have a duplex scan performed.

Owing to litigation patients with saphenopopliteal incompetence should have a duplex scan performed and the
site marked by scan on the day of surgery.

Treatment
Indications for surgery:

• Cosmetic: majority
• Lipodermatosclerosis causing venous ulceration
• Recurrent superficial thrombophlebitis
• Bleeding from ruptured varix
Condition Therapy
Minor varicose veins - no Reassure/ cosmetic therapy
complications
Symptomatic In those without deep venous insufficiency options include foam sclerotherapy,
uncomplicated varicose saphenofemoral / popliteal disconnection, stripping and avulsions, compression
veins stockings
Varicose veins with skin Therapy as above (if compression minimum is formal class I stockings)
changes
Chronic venous Class 2-3 compression stockings (ensure no arterial disease).
insufficiency or ulcers

• Application of formal compression stockings (usually class II/III)

In patients who have suffered ulceration, compression stockings should be worn long term. Where ulceration is
present and established saphenofemoral reflux exists this should be addressed surgically for durable relief of
symptoms, either at the outset or following ulcer healing.

• Injection sclerotherapy (5% Ethanolamine oleate), foam is increasingly popular, though transient
blindness has been reported. Endo venous laser therapy is another minimally invasive option
• Sapheno-femoral or sapheno-popliteal ligation, in the case of the LSV stripping and multiple
phlebectomies

Trendelenburg procedure (sapheno-femoral junction ligation)

• Head tilt 15 degrees and legs abducted


• Oblique incision 1cm medial from artery
• Tributaries ligated (Superficial circumflex iliac vein, Superficial inferior epigastric vein, Superficial and
deep external pudendal vein)
• SF junction double ligated
• Saphenous vein stripped to level of knee/upper calf. NB increased risk of saphenous neuralgia if stripped
more distally

1/3 Question 22-24 of 71


Theme: Management of abdominal aortic aneurysms

A. Immediate laparotomy
B. Immediate CT
C. Elective AAA repair
D. USS in 6 months
E. CT scan during next 4 weeks
F. Endovascular aortic aneurysm repair
G. Discharge
H. Palliate
I. None of the above
Please select the most appropriate management for the scenario given. Each option may be used once, more
than once or not at all.

22. A 66 year old man is referred via the aneurysm screening programme with an abdominal aortic
aneurysm measuring 4.4 cm. Apart from well controlled type 2 DM he is otherwise well

USS in 6 months

At this point continue with ultrasound surveillance

23. A 72 year old man has a CT scan for abdominal discomfort and the surgeon suspects AAA. This
shows a 6.6cm aneurysm with a 3.5cm neck and it continues to involve the right common iliac. The
left iliac is occluded. He is hypertensive and has Type 2 DM which is well controlled.

You answered Endovascular aortic aneurysm repair

The correct answer is Elective AAA repair

Assuming he is fit enough. This would be a typical 'open ' case as the marked iliac disease would
make EVAR difficult

24. An 89 year old man presents with hypotension and collapse and is found by the staff in the toilet of
his care home. He is moribund and unable to give a clear history. He had suffered a cardiac arrest in
the ambulance but has since been resuscitated and now has a Bp of 95 systolic. He has an obviously
palpable AAA.

You answered Immediate laparotomy

The correct answer is Palliate

He will not survive aortic surgery and whilst some may disagree, I would argue that taking this case
to theatre would be futile

Abdominal aorta aneurysm

• Abdominal aortic aneurysms are a common problem in vascular surgery.


• They may occur as either true or false aneurysm. With the former all 3 layers of the arterial wall are
involved, in the latter only a single layer of fibrous tissue forms the aneurysm wall.
• True abdominal aortic aneurysms have an approximate incidence of 0.06 per 1000 people. They are
commonest in elderly men and for this reason the UK is now introducing the aneurysm screening
program with the aim of performing an abdominal aortic ultrasound measurement in all men aged 65
years.

Causes
• Several different groups of patients suffer from aneurysmal disease.
• The commonest group is those who suffer from standard arterial disease, i.e. Those who are
hypertensive, have diabetes and have been or are smokers.
• Other patients such as those suffering from connective tissue diseases such as Marfan's may also
develop aneurysms. In patients with abdominal aortic aneurysms the extracellular matrix becomes
disrupted with a change in the balance of collagen and elastic fibres.

Management

• Most abdominal aortic aneurysms are an incidental finding.


• Symptoms most often relate to rupture or impending rupture.
• 20% rupture anteriorly into the peritoneal cavity. Very poor prognosis.
• 80% rupture posteriorly into the retroperitoneal space
• The risk of rupture is related to aneurysm size, only 2% of aneurysms measuring less than 4cm in
diameter will rupture over a 5 year period. This contrasts with 75% of aneurysms measuring over 7cm in
diameter.
• This is well explained by La Places' law which relates size to transmural pressure.
• For this reason most vascular surgeons will subject patients with an aneurysm size of 5cm or greater to
CT scanning of the chest, abdomen and pelvis with the aim of delineating anatomy and planning
treatment. Depending upon co-morbidities, surgery is generally offered once the aneurysm is between
5.5cm and 6cm.

A CT reconstruction showing an infrarenal abdominal aortic aneurysm. The walls of the sac are calcified which
may facilitate identification on plain x-rays
Image sourced from Wikipedia

Indications for surgery

• Symptomatic aneurysms (80% annual mortality if untreated)


• Increasing size above 5.5cm if asymptomatic
• Rupture (100% mortality without surgery)

Surgical procedures
Abdominal aortic aneurysm repair
Procedure:

GA
Invasive monitoring (A-line, CVP, catheter)
Incision: Midline or transverse
Bowel and distal duodenum mobilised to access aorta.
Aneurysm neck and base dissected out and prepared for cross clamp
Systemic heparinisation
Cross clamp (distal first)
Longitudinal aortotomy
Atherectomy
Deal with back bleeding from lumbar vessels and inferior mesenteric artery
Insert graft either tube or bifurcated depending upon anatomy
Suture using Prolene (3/0 for proximal , distal anastomosis suture varies according to site)
Clamps off: End tidal CO2 will rise owing to effects of reperfusion, at this point major risk of myocardial
events.
Haemostasis
Closure of aneurysm sac to minimise risk of aorto-enteric fistula
Closure: Loop 1 PDS or Prolene to abdominal wall
Skin- surgeons preference

Post operatively:

ITU (Almost all)


Greatest risk of complications following emergency repair
Complications: Embolic- gut and foot infarcts
Cardiac - owing to premorbid states, reperfusion injury and effects of cross clamp
Wound problems
Later risks related to graft- infection and aorto-enteric fistula

Special groups

Supra renal AAA


These patients will require a supra renal clamp and this carries a far higher risk of complications and risk of
renal failure.

Ruptured AAA
Preoperatively the management depends upon haemodynamic instability. In patients with symptoms of rupture
(typical pain, haemodynamic compromise and risk factors) then ideally prompt laparotomy. In those with vague
symptoms and haemodynamic stability the ideal test is CT scan to determine whether rupture has occurred or
not. Most common rupture site is retroperitoneal 80%. These patients will tend to develop retroperitoneal
haematoma. This can be disrupted if Bp is allowed to rise too high so aim for Bp 100mmHg.
Operative details are similar to elective repair although surgery should be swift, blind rushing often makes the
situation worse. Plunging vascular clamps blindly into a pool of blood at the aneurysm neck carries the risk of
injury the vena cava that these patients do not withstand. Occasionally a supracoeliac clamp is needed to effect
temporary control, although leaving this applied for more than 20 minutes tends to carry a dismal outcome.

EVAR
Increasingly patients are now being offered endovascular aortic aneurysm repair. This is undertaken by
surgeons and radiologists working jointly. The morphology of the aneurysm is important and not all are
suitable. Here is a typical list of those features favoring a suitable aneurysm:
• Long neck
• Straight iliac vessels
• Healthy groin vessels

Clearly few AAA patients possess the above and compromise has to be made. The use of fenestrated grafts can
allow supra renal AAA to be treated.

Procedure:

GA
Radiology or theatre
Bilateral groin incisions
Common femoral artery dissected out
Heparinisation
Arteriotomy and insertion of guide wire
Dilation of arteriotomy
Insertion of EVAR Device
Once in satisfactory position it is released
Arteriotomy closed once check angiogram shows good position and no endoleak

Complications:

Endoleaks depending upon site are either Type I or 2. These may necessitate re-intervention and all EVAR
patients require follow up . Details are not needed for MRCS.

Question 25 of 71
During short saphenous vein surgery for varicose veins which of the following nerves is particularly at risk?

A. Sural nerve

B. Popliteal nerve

C. Tibial nerve

D. Femoral nerve

E. Saphenous nerve

Saphenous vein

Long saphenous vein


This vein may be harvested for triple or quadruple bypass surgery

• Originates at the 1st digit where the dorsal vein merges with the dorsal venous arch of the foot
• Passes anterior to the medial malleolus and runs up the medial side of the leg
• At the knee, it runs over the posterior border of the medial epicondyle of the femur bone
• Then passes laterally to lie on the anterior surface of the thigh before entering an opening in the fascia
lata called the saphenous opening
• It joins with the femoral vein in the region of the femoral triangle at the saphenofemoral junction

Tributaries

• Medial marginal
• Superficial epigastric
• Superficial iliac circumflex
• Superficial external pudendal veins

Short saphenous vein

• Originates at the 5th digit where the dorsal vein merges with the dorsal venous arch of the foot, which
attaches to the great saphenous vein.
• It passes around the lateral aspect of the foot (inferior and posterior to the lateral malleolus) and runs
along the posterior aspect of the leg (with the sural nerve)
• Passes between the heads of the gastrocnemius muscle, and drains into the popliteal vein, approximately
at or above the level of the knee joint.

Question 26 of 71
A 21 year old badminton player attends A&E with a painful, swollen right arm. He is right handed. Clinically
he has dusky fingers and his upper limb pulses are present. An axillary vein thrombosis is confirmed. What is
the best acute treatment to achieve vein patency?

A. Intravenous heparin

B. Warfarin

C. Catheter directed tPA

D. Low molecular weight heparin

E. Aspirin

Heparin and warfarin prevent propagation of the clot.

Axillary vein thrombosis

• 1-2% of all deep venous thrombosis


• Primary cause is associated with trauma, thoracic outlet obstruction or repeated effort in a dominant arm
(young active individuals)
• Secondary causes include central line insertion, malignancy, pacemakers
Clinical features

• Pain and swelling (non pitting)


• Numbness
• Discolouration: mottling, dusky
• Pulses present
• Congested veins

Investigations

• FBC: viscosity, platelet function


• Clotting
• Liver function tests
• D-dimer
• Duplex scan: investigation of choice
• CT scan: thoracic outlet obstruction

Treatment

• Local catheter directed TPA


• Heparin
• Warfarin

Question 27 of 71
A 23 year old man presents with a brachial artery embolus. A cervical rib is suspected as being the underlying
cause. From which of the following vertebral levels do they most often arise?

A. C7

B. C5

C. C4

D. C3

E. C2

They usually arise from C7.

Question 28 of 71
A 73 year old man with rest pain and ulceration of the foot undergoes a femoro-distal bypass graft with a PTFE
graft. At the end of the procedure there are good distal foot pulses and a warm pink foot. Over the ensuing 60
days the foot becomes progressively cooler and the pulses diminish. What is the most likely underlying
explanation for this process?
A. Embolus

B. Neo-intimal flap

C. Neo-intimal hyperplasia

D. Polyarteritis

E. Steal syndrome
Neo-intimal hyperplasia in distal arterial anastamoses may be reduced by use of a Miller Cuff when PTFE is
the bypass conduit.

PTFE may induce neo-intimal hyperplasia with subsequent occlusion of the distal anastomosis. In more
proximal arterial bypass surgery the process of neo-intimal hyperplasia is not sufficient to cause anastomotic
occlusion. However, distal bypasses are at greater risk and if vein cannot be used as a conduit then the distal end
of the PTFE should anastomosed to a vein cuff to minimise the risk of neo-intimal hyperplasia.

Anastomoses

• A wide variety of anastomoses are constructed in surgical practice. Essentially the term refers to the
restoration of luminal continuity. As such they are a feature of both abdominal and vascular surgery.

Visceral anastomoses

For an anastomosis to heal three criteria need to be fulfilled:

• Adequate blood supply


• Mucosal apposition
• Minimal tension

When these are compromise the anastomosis may dehisce (leak). Even in the best surgical hands some
anastomoses are more prone to dehiscence than others. Oesophageal and rectal anastomoses are more prone to
leakage and reported leak rates following oesophageal and rectal surgery can be as high as 20%. This figure
includes radiological leaks and those with a clinically significant leak will be of a lower order of magnitude. As
a rule small bowel anastomoses heal most reliably.

The decision as to how best to achieve mucosal apposition is one for each surgeon. Some will prefer the use of
stapling devices as they are quicker to use, others will prefer to perform a sutured anastomosis. The attention to
surgical technique is more important than the method chosen and a poorly constructed stapled anastomosis in
thickened tissue is far more prone to leakage than a hand sewn anastomosis in the same circumstances.

If an anastomosis looks unsafe then it may be best not to construct one at all. In colonic surgery this is relatively
clear cut and most surgeons would bring out an end colostomy. In situations such as oesophageal surgery this is
far more problematic and colonic interposition may be required in this situation.

Vascular anastomoses
Most arterial surgery involving bypasses or aneurysm repairs will require construction of an arterial
anastomosis. Technique is important and for small diameter distal arterial surgery the intimal hyperplasia
resulting from a badly constructed anastomosis may render the whole operation futile before the patient leaves
hospital.

Some key points about vascular anastomoses:

• Always use non absorbable monofilament suture (e.g. Polypropylene).


• Round bodied needle.
• Correct size for anastamosis ( i.e. 6/0 prolene for bottom end of a femoro-distal bypass).
• Suture should be continuous and from inside to outside of artery to avoid raising an intimal flap.

3/3 Question 29-31 of 71


Theme: Lymphoedema Management

A. Homans operation
B. Charles operation
C. Frusemide at high doses
D. Frusemide at low doses
E. Multilayer compression bandaging
F. Lymphovenous anastomosis

Please select the most appropriate management for the lymphoedema scenario given. Each option may be used
once, more than once or not at all.

29. A 52 year old lady develops lower leg swelling following redo varicose vein surgery. There is
evidence of swelling of the left leg up to the knee. The overlying skin appears healthy.

Multilayer compression bandaging

Unfortunately lymphoedema may complicate redo varicose vein surgery (in 0.5% of cases). As the
presentation is mild, she should be managed using compression hosiery. Diuretics do not help in
cases of true lymphoedema and a dramatic response suggests an alternative underlying cause.

30. A 57 year old lady has suffered from lymphoedema for many years. The left leg is swollen to the mid
thigh. Severe limb deformity has developed as a result of process and in spite of compression hosiery.
Lymphoscintography shows no patent lymphatics in the proximal leg. The overlying skin is healthy.

Homans operation

Surgery is indicated in less than 10% of cases. However, severe deformity is one of the indications
for surgery. Lymphovenous anastomosis is indicated where the proximal lymphatics are not patent.
When the overlying skin is healthy (and limb deformity a problem), a Homans procedure is a
reasonable first line operative option.

31. A 38 year old lady is troubled by lymphoedema that occurred following a block dissection of the
groin for malignant melanoma many years previously. Despite therapy with compression bandages
she has persistent lower leg swelling impairing her activities of daily living. She has no evidence of
recurrent malignancy. Lymphoscintography demonstrates occlusion of the groin lymphatics.
However, the distal lymphatic system appears healthy.

Lymphovenous anastomosis

In young patients with proximal disease and healthy distal lymphatics a lymphovenous anastomosis
may be considered. Such cases are rare.

Lymphoedema

• Due to impaired lymphatic drainage in the presence of normal capillary function.


• Lymphoedema causes the accumulation of protein rich fluid, subdermal fibrosis and dermal thickening.
• Characteristically fluid is confined to the epifascial space (skin and subcutaneous tissues); muscle
compartments are free of oedema. It involves the foot, unlike other forms of oedema. There may be a
'buffalo hump' on the dorsum of the foot and the skin cannot be pinched due to subcutaneous fibrosis.

Causes of lymphoedema

Primary • Congenital < 1 year: sporadic, Milroy's disease


• Onset 1-35 years: sporadic, Meige's disease
• > 35 years: Tarda

Secondary • Bacterial/fungal/parasitic infection (filariasis)


• Lymphatic malignancy
• Radiotherapy to lymph nodes
• Surgical resection of lymph nodes
• DVT
• Thrombophlebitis

Indications for surgery

• Marked disability or deformity from limb swelling


• Lymphoedema caused by proximal lymphatic obstruction with patent distal lymphatics suitable for a
lymphatic drainage procedure
• Lymphocutaneous fistulae and megalymphatics

Procedures
Homans operation Reduction procedure with preservation of overlying skin (which must be in good
condition). Skin flaps are raised and the underlying tissue excised. Limb circumference
typically reduced by a third.
Charles operation All skin an subcutaneous tissue around the calf is excised down to the deep fascia. Split
skin grafts are placed over the site. May be performed if overlying skin is not in good
condition. Larger reduction in size than with Homans procedure.
Lymphovenous Identifiable lymphatics are anastomosed to sub dermal venules. Usually indicated in 2% of
anastamosis patients with proximal lymphatic obstruction and normal distal lymphatics.

Question 32 of 71
Which of the following is not a typical feature of a chronic venous leg ulcer?

A. Heaped raised borders if the ulcer has been present more than 5 years

B. Evidence of surrounding lipodermatosclerosis

C. Irregular shape to the ulcer

D. 20% of cases will have a previous history of deep vein thrombosis

E. Haemosiderin deposits in surrounding skin

The borders of the ulcer are often well defined even though they may be irregular. Heaped or raised borders
should raise suspicion of a marjolins ulcer.

Question 33 of 71
Which of the following is not a typical feature of an arterial leg ulcer?

A. Well demarcated edges

B. A grey - white base to the ulcer

C. Men are affected more than women

D. Painful

E. Ankle swelling

Ankle swelling is often absent. If present it may be due to mixed arteriovenous disease. With mixed disease the
arterial component is treated first.

Peripheral vascular disease

Indications for surgery to revascularise the lower limb

• Intermittent claudication
• Critical ischaemia
• Ulceration
• Gangrene
Intermittent claudication that is not disabling may provide a relative indication, whilst the other complaints are
often absolute indications depending upon the frailty of the patient.

Assessment

• Clinical examination
• Ankle brachial pressure index measurement
• Duplex arterial ultrasound
• Angiography (standard, CT or MRI): usually performed only if intervention being considered.

Angioplasty
In order for angioplasty to be undertaken successfully the artery has to be accessible. The lesion relatively short
and reasonable distal vessel runoff. Longer lesions may be amenable to sub-intimal angioplasty.

Surgery
Surgery will be undertaken where attempts at angioplasty have either failed or are unsuitable. Bypass essentially
involves bypassing the affected arterial segment by utilising a graft to run from above the disease to below the
disease. As with angioplasty good runoff improves the outcome.

Some key concepts with bypass surgery

Superficial femoral artery occlusion to the above knee

• Angioplasty may be attempted but otherwise these patients will require a femoro-popliteal bypass graft.
• Patency rates for Polytetrafluoroethylene (PTFE) and vein are similar, so PTFE preferred unless co-
existing infection makes use of prosthetic material undesirable.

Procedure

• Artery dissected out, IV heparin 3,000 units given and then the vessels are cross clamped
• Longitudinal arteriotomy
• Graft cut to size and tunneled to arteriotomy sites
• Anastomosis to femoral artery usually with 5/0 'double ended' Prolene suture
• Distal anastomosis usually using 6/0 'double ended' Prolene

Distal disease

• Femoro-distal bypass surgery takes longer to perform, is more technically challenging and has higher
failure rates.
• In elderly diabetic patients with poor runoff a primary amputation may well be a safer and more
effective option. There is no point in embarking on this type of surgery in patients who are wheelchair
bound.
• In femorodistal bypasses vein gives superior outcomes to PTFE.

Rules
• Vein mapping 1st to see whether there is suitable vein (the preferred conduit). Sub intimal hyperplasia
occurs early when PTFE is used for the distal anastomosis and will lead to early graft occlusion and
failure.
• Essential operative procedure as for above knee fem-pop.
• If there is insufficient vein for the entire conduit then vein can be attached to the end of the PTFE graft
and then used for the distal anastomosis. This type of 'vein boot' is technically referred to as a Miller
Cuff and is associated with better patency rates than PTFE alone.
• Remember the more distal the arterial anastomosis the lower the success rate.

3/3 Question 34-36 of 71


Theme: Lower limb ulceration

A. Mixed ulcer
B. Chronic obliterative arterial disease
C. Superficial venous insufficiency
D. Deep venous insufficiency
E. Neuropathic ulcer
F. Basal cell carcinoma
G. Squamous cell carcinoma

Please select the most likely cause of ulceration for the scenario given. Each option may be used once, more
than once or not at all.

34. A 65 year old diabetic female presents with a painless ulcer at the medial malleolus, it has been
present for the past 16 years. On examination she has evidence of truncal varicosities and a brownish
discolouration of the skin overlying the affected area.

Superficial venous insufficiency

Theme from September 2012 Exam


Venous ulcers are usually associated with features of venous insufficiency. These include
haemosiderin deposition and varicose veins. Neuropathic ulcers will tend to present at sites of
pressure, which is not typically at the medial malleolus.

35. A 71 year old man presents with a painful lower calf ulcer, mild pitting oedema and an ABPI of 0.3.

Chronic obliterative arterial disease

Painful ulcers associated with a low ABPI are usually arterial in nature. The question does not
indicate that features of chronic venous insufficiency are present. Patients may have mild pitting
oedema as many vascular patients will also have ischaemic heart disease and elevated right heart
pressures. The absence of more compelling signs of venous insufficiency makes a mixed ulcer less
likely.
36. A 79 year old retired teacher has had an ulcer for 15 years. It is at the medial malleolus and has
associated lipodermatosclerosis of the lower limb. The ulcer base is heaped up and irregular.

Squamous cell carcinoma

If after many years an ulcer becomes heaped up and irregular, with rolled edges then suspect a
squamous cell carcinoma.

Question 37 of 71
A 66 year old man is admitted with severe angina. There is a lesion of the proximal left anterior descending
coronary artery. Which of the following would be the most suitable conduit for bypass?

A. Long saphenous vein

B. Short saphenous vein

C. Cephalic vein

D. Internal mammary artery

E. Thoraco-acromial artery

The internal mammary artery is an excellent conduit for coronary artery bypass. It has better long term patency
rates than venous grafts. The thoraco-acromial artery is seldom used.

Cardiopulmonary bypass

Indications for surgery

• Left main stem stenosis or equivalent (proximal LAD and proximal circumflex)
• Triple vessel disease
• Diffuse disease unsuitable for PCI

The guidelines state that CABG is the preferred treatment in high-risk patients with severe ventricular
dysfunction or diabetes mellitus.

Technique
General anaesthesia
Central and arterial lines
Midline sternotomy or left sub mammary incision
Aortic root and pericardium dissected
Heart inspected

Bypass grafting may be performed using a cardiopulmonary bypass circuit with cardiac arrest or using a number
of novel 'off pump' techniques.
Procedure cardiopulmonary bypass

• Aortic root cannulated


• Right atrial cannula
• Circuit primed and patient fully heparinised (30,000 Units unfractionated heparin) as the circuit is highly
thrombogenic
• Flow established through circuit
• Aortic cross clamp applied
• Cardioplegia solution instilled into the aortic root below cross clamp
• Heart now asystolic and ready for surgery

Off pump techniques are evolving on a constant basis and details are beyond the scope of the MRCS.

Conduits for bypass


> Internal mammary artery is best. Use of both is associated with increased risk of sternal wound dehiscence.
However, many surgeons will use both especially for redo surgery.
> Radial artery harvested from forearm. Ensure ulnar collateral working first!
> Reversed long saphenous vein grafts
Typically anastamosed using 7/0-8/0 prolene sutures (distally) and 6/0 prolene for top end.

Once flow established


Anticoagulation reversed using protamine
Patient is taken off bypass
Inotropes given if needed
Sternum closed using sternal closure device or stainless steel wire

Complications

• Post perfusion syndrome: transient cognitive impairment


• Non union of the sternum; due to loss of the internal thoracic artery
• Myocardial infarction
• Late graft stenosis
• Acute renal failure
• Stroke
• Gastrointestinal

Perioperative risk is quantified using the Parsonnet and Euroscores and unit outcomes are audited using this
data.

Question 38 of 71
Concerning proximal aortic dissection (Debakey types 1 and 2/ Stanford type A) which statement is false?

A. The intimal tear is typically >50% of the aortic circumference.

B. It is usually treated using an endovascular approach.


C. They have a 50% mortality in the first 2 days.

D. Arch reconstructions may require deep hypothermic circulatory


arrest.

E. Target systolic pressure of <110mmHg should be maintained.

Usually open surgery is required for these lesions as customised grafts are not usually available for this type of
repair yet.

Aortic dissection

• More common than rupture of the abdominal aorta


• 33% of patients die within the first 24 hours, and 50% die within 48 hours if no treatment received
• Associated with hypertension
• Features of aortic dissection: tear in the intimal layer, followed by formation and propagation of a
subintimal hematoma. Cystic medial necrosis (Marfan's)
• Most common site of dissection: 90% occurring within 10 centimetres of the aortic valve

Stanford Classification
Type Location Treatment
A Ascending aorta/ aortic root Surgery- aortic root replacement
B Descending aorta Medical therapy with antihypertensives

DeBakey classification
Type Site affected
I Ascending aorta, aortic arch, descending aorta
II Ascending aorta only
III Descending aorta distal to left subclavian artery

Clinical features

• Tearing, sudden onset chest pain (painless 10%)


• Hypertension or Hypotension
• A blood pressure difference greater than 20 mm Hg
• Neurologic deficits (20%)

Investigations

• CXR: widened mediastinum, abnormal aortic knob, ring sign, deviation trachea/oesophagus
• CT (spiral)
• MRI
• Angiography (95% of patients diagnosed)

Management
• Beta-blockers: aim HR 60-80 bpm and systolic BP 100-120 mm Hg.
• Urgent surgical intervention: type A dissections. This will usually involve aortic root replacement.

Question 39 of 71
A 67 year old male is diagnosed as having a 7cm infra renal abdominal aortic aneurysm. What is the likely risk
of rupture over the next 5 years?

A. <10%

B. 20%

C. 25%

D. 75%

E. 35%
Risks of abdominal aortic aneurysm rupture
(over 5 years):

• 5-5.9cm = 25%
• 6-6.9cm= 35%
• 7cm and over = 75%

Aneuryms greater than 5cm in diameter on USS should be formally assessed using CT scanning with arterial
phases to delineate anatomy and facilitate surgical planning.

1/3 Question 40-42 of 71


Theme: Venous disease

A. No further management needed


B. Injection sclerotherapy 0.5% Sodium tetradecyl sulphate
C. Injection sclerotherapy 5% phenol
D. Long saphenous vein ligation
E. Long saphenous vein stripped to the ankle
F. Long saphenous vein stripped to the knee
G. Doppler scan
H. Duplex scan

Please select the most appropriate management plan for the scenario given. Each option may be used once,
more than once or not at all.
40. A 42 year old teacher presents with an ulcer associated with varicose veins in the long saphenous vein
territory. Apart from a DVT 1 year ago, she has no other past medical history.

Duplex scan

This patient needs a duplex scan to assess the patency of her deep venous system before surgery can
be undertaken. Other indications for duplex scan include recurrent varicose veins or complications.

41. A 42 year old accountant presents with thrombophlebitis of a long standing varicosity of the inner
thigh. Doppler and clinical assessment demonstrate saphenofemoral junction incompetence.

You answered Long saphenous vein ligation

The correct answer is Long saphenous vein stripped to the knee

Long standing varicose veins with complications such as thrombophlebitis and demonstrated valvular
incompetence should receive surgery. Ligation was previously popular but has higher long term
recurrence rates. There is increased risk of saphenous neuralgia if they are stripped to the ankle.

42. A 28 year old lady presents with increasing aching discomfort from a varicosity below the knee. On
examination she has a single large truncal varicosity in the area, doppler assessment shows competent
sapheno-femoral and sapheno-popliteal junctions.

You answered Long saphenous vein stripped to the ankle

The correct answer is Injection sclerotherapy 0.5% Sodium tetradecyl sulphate

This lady has a single varicosity. Sclerotherapy (probably with foam) would be the ideal
management. A simple avulsion under anaethesia is an alternative.
These are best treated with injection sclerotherapy. 5% phenol is normally used as a sclerosing agent
in haemorrhoids!

Chronic venous insufficiency and Varicose veins

Wide spectrum of disease ranging from minor cosmetic problem through to ulceration and disability. It is
commoner in women than men and is worse during pregnancy.

• Defined as saccular dilatation of veins (WHO)

The veins of the lower limb consist of an interconnected network of superficial and deep venous systems.
Varices occur because of localised weakness in the vein wall resulting in dilatation and reflux of blood due to
non union of valve cusps.

• Histology: fibrous scar tissue dividing smooth muscle within media in the vessel wall
Tissue damage in chronic venous insufficiency occurs because of perivascular cytokine leakage resulting in
localised tissue damage coupled with impaired lymphatic flow.

• Affected veins: normally long and short saphenous veins

Diagnosis
Typical symptoms of varicose veins include:

• Cosmetic appearance
• Aching
• Ankle swelling that worsens as the day progresses
• Episodic thrombophlebitis
• Bleeding
• Itching

Symptoms of chronic venous insufficiency include:

• Dependant leg pain


• Prominent leg swelling
• Oedema extending beyond the ankle
• Venous stasis ulcers

The typical venous stasis ulcer is:

• Located above the medial malleolus


• Indolent appearance with basal granulation tissue
• Variable degree of scarring
• Non ischaemic edges
• Haemosiderin deposition in the gaiter area (and also lipodermatosclerosis).

Differential diagnosis

• Lower limb arterial disease


• Marjolins ulcer
• Claudication
• Spinal stenosis
• Swelling due to medical causes e.g. CCF.

Exclusion of these differentials is by means of physical examination and ankle brachial pressure index
measurement.

Examination
• Assess for dilated short saphenous vein (popliteal fossa) and palpate for saphena varix medial to the
femoral artery
• Brodie-Trendelenburg test: to assess level of incompetence
• Perthes' walking test: assess if deep venous system competent

Investigation

• Doppler exam: if incompetent a biphasic signal due to retrograde flow is detected


• Duplex scanning: to ensure patent deep venous system (do if DVT or trauma)

All patients should have a Doppler assessment to assess for venous reflux and should be classified as having
uncomplicated varicose veins or varicose veins with associated chronic venous insufficiency. In the history
establishing a previous thrombotic event (DVT/ lower limb fracture) is important and patients with such a
history and all who have evidence of chronic venous insufficiency should have a duplex scan performed.

Owing to litigation patients with saphenopopliteal incompetence should have a duplex scan performed and the
site marked by scan on the day of surgery.

Treatment
Indications for surgery:

• Cosmetic: majority
• Lipodermatosclerosis causing venous ulceration
• Recurrent superficial thrombophlebitis
• Bleeding from ruptured varix

Condition Therapy
Minor varicose veins - no Reassure/ cosmetic therapy
complications
Symptomatic In those without deep venous insufficiency options include foam sclerotherapy,
uncomplicated varicose saphenofemoral / popliteal disconnection, stripping and avulsions, compression
veins stockings
Varicose veins with skin Therapy as above (if compression minimum is formal class I stockings)
changes
Chronic venous Class 2-3 compression stockings (ensure no arterial disease).
insufficiency or ulcers

• Application of formal compression stockings (usually class II/III)

In patients who have suffered ulceration, compression stockings should be worn long term. Where ulceration is
present and established saphenofemoral reflux exists this should be addressed surgically for durable relief of
symptoms, either at the outset or following ulcer healing.

• Injection sclerotherapy (5% Ethanolamine oleate), foam is increasingly popular, though transient
blindness has been reported. Endo venous laser therapy is another minimally invasive option
• Sapheno-femoral or sapheno-popliteal ligation, in the case of the LSV stripping and multiple
phlebectomies

Trendelenburg procedure (sapheno-femoral junction ligation)

• Head tilt 15 degrees and legs abducted


• Oblique incision 1cm medial from artery
• Tributaries ligated (Superficial circumflex iliac vein, Superficial inferior epigastric vein, Superficial and
deep external pudendal vein)
• SF junction double ligated
• Saphenous vein stripped to level of knee/upper calf. NB increased risk of saphenous neuralgia if stripped
more distally

Question 43 of 71
A 24-year-old female is referred to the acute surgical team as she is noted to have an absent left radial pulse.
Apart from some dizziness and lethargy, the patient does not have any features suggestive of an acute ischaemic
limb. Blood tests are as follows:

Na+ 136 mmol/l


K+ 4.1 mmol/l
Urea 2.3 mmol/l
Creatinine 77 µmol/l

ESR 66 mm/hr

What is the most likely diagnosis?

A. Turner's syndrome

B. Takayasu's arteritis

C. Kawasaki disease

D. Coarctation of the aorta

E. Breast carcinoma with local spread

Takayasu's arteritis

Takayasu's arteritis is a large vessel vasculitis. It typically causes occlusion of the aorta and questions
commonly refer to an absent limb pulse. It is more common in females and Asian people

Associations
• renal artery stenosis

Management

• steroids

2/3 Question 44-46 of 71


Theme: Leg swelling

A. Milroy's disease
B. Meige's disease
C. Lymphoedema tarda
D. Filariasis
E. Tuberculosis
F. Locally advanced bladder carcinoma
G. Malaria

Which is the most likely diagnosis for the scenario given? Each option may be used once, more than once or not
at all.

44. The medical team refer a 72 year old lady with a bilateral swollen legs. Deep vein thrombosis has
been excluded and there is no response to diuretics. On further questioning, the patient reveals that
she was born with the swelling in both of her legs.

Milroy's disease

Milroy's disease is present from birth and is due to failure of the lymphatic vessels to develop. Note
that Meige's disease develops AFTER birth.

45. A 52 year old woman presents with rapid swelling of the left leg. The swelling is greater in the thigh
compared to the calf.

You answered Lymphoedema tarda

The correct answer is Locally advanced bladder carcinoma

Always consider a malignancy in an older adult with new lymphoedema in a limb, especially if the
swelling is greater proximally than distally. If malignancy is excluded consider the diagnosis of
lymphoedema tarda.

46. A 34 year old African teacher attends A&E with a swollen leg. She has been in England for 2 weeks.
She lives in an area prevalent with mosquitoes and where there is poor sanitation.
Filariasis

Filariasis is caused by the nematode Wucheria bancrofti, which is mainly spread by mosquito. The
oedema can be gross leading to elephantitis. Treatment is with diethylcarbamazine.

Lymphoedema

• Due to impaired lymphatic drainage in the presence of normal capillary function.


• Lymphoedema causes the accumulation of protein rich fluid, subdermal fibrosis and dermal thickening.
• Characteristically fluid is confined to the epifascial space (skin and subcutaneous tissues); muscle
compartments are free of oedema. It involves the foot, unlike other forms of oedema. There may be a
'buffalo hump' on the dorsum of the foot and the skin cannot be pinched due to subcutaneous fibrosis.

Causes of lymphoedema

Primary • Congenital < 1 year: sporadic, Milroy's disease


• Onset 1-35 years: sporadic, Meige's disease
• > 35 years: Tarda

Secondary • Bacterial/fungal/parasitic infection (filariasis)


• Lymphatic malignancy
• Radiotherapy to lymph nodes
• Surgical resection of lymph nodes
• DVT
• Thrombophlebitis

Indications for surgery

• Marked disability or deformity from limb swelling


• Lymphoedema caused by proximal lymphatic obstruction with patent distal lymphatics suitable for a
lymphatic drainage procedure
• Lymphocutaneous fistulae and megalymphatics

Procedures
Homans operation Reduction procedure with preservation of overlying skin (which must be in good
condition). Skin flaps are raised and the underlying tissue excised. Limb circumference
typically reduced by a third.
Charles operation All skin an subcutaneous tissue around the calf is excised down to the deep fascia. Split
skin grafts are placed over the site. May be performed if overlying skin is not in good
condition. Larger reduction in size than with Homans procedure.
Lymphovenous Identifiable lymphatics are anastomosed to sub dermal venules. Usually indicated in 2% of
anastamosis patients with proximal lymphatic obstruction and normal distal lymphatics.
Question 47 of 71
A 21 year old post man notices leg pain after 5 minutes walking during his round. It improves 3 minutes after
stopping. Clinically he is noted to have reduced hair of the lower limbs and his calf muscles appear atrophied.
There is a weak popliteal pulse, but it is still present when the knee is fully extended. What is the most likely
diagnosis?

A. Occlusive arterial disease caused by atherosclerosis

B. Popliteal fossa entrapment

C. Cerebral vascular accident

D. Diabetes mellitus

E. Adductor canal compression syndrome

Adductor canal compression syndrome most commonly presents in young males and is an important
differential diagnosis in men presenting with symptoms of acute limb ischaemia on exertion. It is caused by
compression of the femoral artery by the musculotendinous band from adductor magnus muscle.
The treatment consists of the division of the abnormal band and restoration of the arterial circulation. Popliteal
fossa entrapment is the main differential diagnosis, however the pulse disappears when the knee is fully
extended.

Adductor canal

• Also called Hunter's or subsartorial canal

• Immediately distal to the apex of the femoral triangle, lying in the middle third of the thigh. Canal
terminates at the adductor hiatus.

Borders Contents
Laterally Vastus medialis muscle Saphenous nerve
Posteriorly Adductor longus, adductor Superficial femoral
magnus artery
Roof Sartorius Superficial femoral vein
0/3 Question 48-50 of 71
Theme: Peripheral arterial disease

A. Femoro-above knee popliteal bypass graft using PTFE


B. Femoro-above knee popliteal bypass graft using long saphenous vein
C. Femoro-distal bypass graft using PTFE
D. Femoro-distal bypass graft using PTFE with Miller Cuff
E. Femoro-distal bypass graft using long saphenous vein
F. Above knee amputation
G. Below knee amputation
H. Angioplasty

Please select the most appropriate management for the vascular scenario given. Each option may be used
once, more than once or not at all.

48. A 63 year old man presents with a non healing ulcer on his foot. His ABPI measurements are 0.35.
A duplex scan shows a 4 cm 90% stenotic lesion of the superficial femoral artery with no distal
disease.

You answered Femoro-above knee popliteal bypass graft using long saphenous vein

The correct answer is Angioplasty

Short occlusions are generally reasonable candidates for primary attempts at angioplasty.

49. A 63 year old man who previously smoked 20 cigarettes a day and has newly diagnosed type II
diabetes. He presents with rest pain. A diagnostic angiogram demonstrates occlusion of the distal
superficial femoral artery continuing below the knee. He has reasonable posterior tibial artery
below this level which branches to give good runoff to the foot. He has varicose veins.

You answered Femoro-distal bypass graft using long saphenous vein

The correct answer is Femoro-distal bypass graft using PTFE with Miller Cuff

This man needs a bypass operation. Using PTFE alone will not give a good result as sub intimal
hyperplasia will give poor outcome early. Using a vein cuff (Miller cuff) at the end of a PTFE graft
will improve the situation.

50. A wheelchair bound 78 year old women with ischaemic heart disease secondary to long smoking
history and longstanding type II diabetes presents with rest pain and a non healing ulcer on the
dorsum of her foot. Angiogram shows reasonable superficial femoral artery and iliacs. At the level
of the popliteal there is an occlusion. Below this there is a short area of patent posterior tibial and
this reconstitutes lower down the leg to flow to the foot.

You answered Femoro-distal bypass graft using PTFE

The correct answer is Above knee amputation

A femoro-distal bypass graft would carry a high risk of failure and risk of peri-operative myocardial
infarct. This lady would be well suited to primary amputation.

Peripheral vascular disease

Indications for surgery to revascularise the lower limb

• Intermittent claudication
• Critical ischaemia
• Ulceration
• Gangrene
Intermittent claudication that is not disabling may provide a relative indication, whilst the other complaints
are often absolute indications depending upon the frailty of the patient.

Assessment

• Clinical examination
• Ankle brachial pressure index measurement
• Duplex arterial ultrasound
• Angiography (standard, CT or MRI): usually performed only if intervention being considered.

Angioplasty
In order for angioplasty to be undertaken successfully the artery has to be accessible. The lesion relatively
short and reasonable distal vessel runoff. Longer lesions may be amenable to sub-intimal angioplasty.

Surgery
Surgery will be undertaken where attempts at angioplasty have either failed or are unsuitable. Bypass
essentially involves bypassing the affected arterial segment by utilising a graft to run from above the disease
to below the disease. As with angioplasty good runoff improves the outcome.

Some key concepts with bypass surgery

Superficial femoral artery occlusion to the above knee

• Angioplasty may be attempted but otherwise these patients will require a femoro-popliteal bypass
graft.
• Patency rates for Polytetrafluoroethylene (PTFE) and vein are similar, so PTFE preferred unless co-
existing infection makes use of prosthetic material undesirable.

Procedure

• Artery dissected out, IV heparin 3,000 units given and then the vessels are cross clamped
• Longitudinal arteriotomy
• Graft cut to size and tunneled to arteriotomy sites
• Anastomosis to femoral artery usually with 5/0 'double ended' Prolene suture
• Distal anastomosis usually using 6/0 'double ended' Prolene

Distal disease

• Femoro-distal bypass surgery takes longer to perform, is more technically challenging and has higher
failure rates.
• In elderly diabetic patients with poor runoff a primary amputation may well be a safer and more
effective option. There is no point in embarking on this type of surgery in patients who are wheelchair
bound.
• In femorodistal bypasses vein gives superior outcomes to PTFE.

Rules
• Vein mapping 1st to see whether there is suitable vein (the preferred conduit). Sub intimal hyperplasia
occurs early when PTFE is used for the distal anastomosis and will lead to early graft occlusion and
failure.
• Essential operative procedure as for above knee fem-pop.
• If there is insufficient vein for the entire conduit then vein can be attached to the end of the PTFE graft
and then used for the distal anastomosis. This type of 'vein boot' is technically referred to as a Miller
Cuff and is associated with better patency rates than PTFE alone.
• Remember the more distal the arterial anastomosis the lower the success rate.

Question 51 of 71
Which of the following is not a feature of a Charcot foot?

A. Bounding foot pulses in


the early phases

B. Often occurs in the


complete absence of
trauma

C. Erythema of the foot in


the early phase

D. Autonomic neuropathy

E. Peripheral neuropathy

Do not confuse the early phase of


Charcot foot with cellulitis

Trauma (even if only minor) is a prerequisite. Patients cannot usually recall the
traumatic event. The associated neuropathy means that patients continue to walk
on the affected foot with subsequent deformity developing over time.

Question 52 of 71
A 34 year old man presents with varicose veins and it is suspected that these are part of the Klippel-Trenaunay
syndrome. Which of the following is not a characteristic of this condition?

A. Presence of varicose veins

B. Gigantism of a limb

C. Long saphenous vein involvement

D. Port wine stains with clear borders

E. Arteriovenous fistulae
The Klippel-Trenaunay vein is a large, lateral, superficial vein sometimes seen at birth. This vein begins in the
foot or the lower leg and travels proximally until it enters the thigh or the gluteal area. Otherwise, varicosities
may not be clinically evident until the child begins to ambulate.
Varicosities may be extensive, though they often spare the saphenous distribution. They are seen below the
knee, laterally above the knee, and occasionally in the pelvic region. Varicosities may affect the superficial,
deep, and perforating venous systems.
Surgical exploration has demonstrated atresia and agenesis of deep veins, compression due to fibrous bands,
aberrant arteries, abnormal muscles, or venous sheaths.
Rarely, varicosities have been found in the bladder, the colon, and the pulmonary vessels

Klippel-Trenaunay syndrome

Klippel-Trenaunay-Weber syndrome generally affects a single extremity, although cases of multiple affected
limbs have been reported. The leg is the most common site followed by the arms, the trunk, and rarely the
head and the neck

Signs and symptoms


The birth defect is diagnosed by the presence of a combination of these symptoms:

• One or more distinctive port-wine stains with sharp borders


• Varicose veins
• Hypertrophy of bony and soft tissues, that may lead to local gigantism or shrinking.
• An improperly developed lymphatic system

In some cases, port-wine stains (capillary port wine type) may be absent. Such cases are very rare and may be
classified as "atypical Klippel-Trenaunay syndrome".

KTS can either affect blood vessels, lymph vessels, or both. The condition most commonly presents with a
mixture of the two. Those with venous involvement experience increased pain and complications.

1/3 Question 53-55 of 71


Theme: Diabetic foot sepsis

A. IV broad spectrum antibiotics


B. Incision and drainage of pus
C. Ray amputation
D. Below knee amputation
E. Above knee amputation
F. Vacuum Assisted Closure device (VAC)
G. Discharge home
H. Application of 4 layer bandages

Please select the most appropriate management for the scenario given. Each option may be used once, more
than once or not at all.

53. A 68 year old man with type II diabetes has a non healing ulcer following a ray amputation 2 weeks
ago. An x-ray shows no osteomyelitis and the ABPI is >1.

You answered Discharge home

The correct answer is Vacuum Assisted Closure device (VAC)

A VAC dressing may avoid the need for further surgery.

54. A 48 year old woman is admitted with sepsis secondary to an infected diabetic foot ulcer. She has a
necrotic and infected forefoot with necrosis of the heel. There is a boggy indurated swelling anterior
to the ankle joint. Pulses are normal.

You answered Incision and drainage of pus

The correct answer is Below knee amputation

A below knee amputation is the best option here. The foot is non salvageable. However, she may
ambulate with a prosthesis.

55. An 84 year old lady is admitted with an infected diabetic foot. An x-ray shows osteomyelitis of her
calcaneum. She has a fixed flexion deformity of her knee, but normal pulses.

Above knee amputation

This patient will not be able to walk with a below knee amputation, therefore an above knee
amputation would be preferable, as it guarantees better healing the short term.
Consider above knee amputation in patients with fixed
flexion deformity.

Amputations

Amputations are indicated when the affected limb is one of the following:

• Dead non viable


• Deadly where it is posing a major threat to life
• Dead useless where it is viable but a prosthesis would be preferable

Orthopaedic surgery

• Amputation is often undertaken as an option of last resort e.g. Limb salvage has failed and the limb is
so non functional that mobility needs would be best met with prosthesis.
• Chronic fracture non union or significant limb shortening following trauma would fit into this
category. Occasionally following major trauma a primary amputation is preferable. This would be the
case in an open fracture with major distal neurovascular compromise and other more life threatening
injuries are present.

Vascular surgery

• The first two categories are the most prevalent.


• Diabetic foot sepsis is often a major cause of sepsis which can spread rapidly in the presence of
established peripheral vascular disease.
• As a general rule the main issue in vascular surgery is to optimise vascular inflow prior to surgery. The
more distal the planned amputation is to be, the more important this rule becomes.
• In other situations there has been something such as an embolic event that has not been revascularised
in time. In this case the limb shows fixed mottling and an amputation will be needed.

Types of amputations
As the vast majority of commonly performed amputations affect the lower limbs these will be covered here.

The main categories of amputations are:

• Pelvic disarticulation (hindquarter)


• Above knee amputation
• Gritti Stokes (through knee amputation)
• Below knee amputation (using either Skew or Burgess flaps)
• Syme's amputation (through ankle)
• Amputations of mid foot and digits

Choosing a level of amputation depends on:

• The disease process being treated


• Desired functional outcome
• Co-morbidities of the patient

Above knee amputations

• Quick to perform
• Heal reliably
• Patients regain their general health quickly
• For this benefit, a functional price has to be paid and many patients over the age of 70 will never walk
on an above knee prosthesis.
• Above knee amputations use equal anterior-posterior flaps

Below knee amputations

• Technically more challenging to perform


• Heal less reliably than their above knee counterparts.
• However, many more patients are able to walk using a below knee prosthesis.
• In below knee amputations the two main flaps are Skew flaps or the Burgess
Long posterior flap. There is some evidence that Skew flaps are better
vascularised than the long posterior flap and some vascular surgeons prefer
them for this reason.

It is worth remembering that whilst it may be technically feasible to offer a below knee amputation there may
be circumstances where an above knee option is preferable. For example, in fixed flexion deformities of the
lower limb, little functional benefit would be gained from below knee amputation surgery.

Question 56 of 71
Which of the following statements related to coronary artery bypass surgery is true?

A. Late graft stenosis is mainly associated with saphenous vein grafts

B. Is indicated if there is stenosis > 70% of the right coronary artery

C. The left atrium is cannulated during the procedure

D. The CHADS score is used to assess peri operative risk

E. Cardioplegia is always undertaken at a 37 degrees

Indications are:

1. Left main stem stenosis or equivalent (proximal LAD and proximal circumflex)
2. Triple vessel disease
3. Diffuse disease unsuitable for PCI
The right atrium is cannulated. The CHADS score assesses whether a patient should be warfarinised if they
have atrial fibrillation. Cardioplegia can be undertaken at cold or warm temperatures.

Cardiopulmonary bypass

Indications for surgery

• Left main stem stenosis or equivalent (proximal LAD and proximal circumflex)
• Triple vessel disease
• Diffuse disease unsuitable for PCI

The guidelines state that CABG is the preferred treatment in high-risk patients with severe ventricular
dysfunction or diabetes mellitus.

Technique
General anaesthesia
Central and arterial lines
Midline sternotomy or left sub mammary incision
Aortic root and pericardium dissected
Heart inspected

Bypass grafting may be performed using a cardiopulmonary bypass circuit with cardiac arrest or using a
number of novel 'off pump' techniques.

Procedure cardiopulmonary bypass

• Aortic root cannulated


• Right atrial cannula
• Circuit primed and patient fully heparinised (30,000 Units unfractionated heparin) as the circuit is
highly thrombogenic
• Flow established through circuit
• Aortic cross clamp applied
• Cardioplegia solution instilled into the aortic root below cross clamp
• Heart now asystolic and ready for surgery

Off pump techniques are evolving on a constant basis and details are beyond the scope of the MRCS.

Conduits for bypass


> Internal mammary artery is best. Use of both is associated with increased risk of sternal wound dehiscence.
However, many surgeons will use both especially for redo surgery.
> Radial artery harvested from forearm. Ensure ulnar collateral working first!
> Reversed long saphenous vein grafts
Typically anastamosed using 7/0-8/0 prolene sutures (distally) and 6/0 prolene for top end.

Once flow established


Anticoagulation reversed using protamine
Patient is taken off bypass
Inotropes given if needed
Sternum closed using sternal closure device or stainless steel wire

Complications

• Post perfusion syndrome: transient cognitive impairment


• Non union of the sternum; due to loss of the internal thoracic artery
• Myocardial infarction
• Late graft stenosis
• Acute renal failure
• Stroke
• Gastrointestinal

Perioperative risk is quantified using the Parsonnet and Euroscores and unit outcomes are audited using this
data.

Question 57 of 71
A 73 year old man develops sudden onset abdominal pain and collapses. On examination he has a tender
pulsatile mass in his upper abdomen. He has a blood pressure of 90/60mmHg and pulse rate of 105 beats per
minute. Which of the following intravenous fluid regimens is most appropriate, whilst waiting for operative
repair?
A. 1 Litre of pentastarch over 15 minutes

B. 1 litre of Hartmans solution over 4 hours

C. 1 litre of gelofusin over 30 minutes

D. 1 litre of Hartmans solution over 30 minutes

E. 1 litre of blood over 15 minutes

This man will have a contained haematoma and is awaiting surgery. Rapid, high volume infusions may cause
this to dislodge with disastrous consequences

1/3 Question 58-60 of 71


Theme: Mesenteric vascular disease

A. Abdominal ultrasound
B. Abdominal CT with venous phase contrast
C. CT angiogram
D. Magnetic resonance angiogram
E. Duplex ultrasound
F. Abdominal x-ray
G. Meseneric angiography via brachial artery

Please select the most appropriate investigation for the scenario given. Each option may be used once, more
than once or not at all.

58. A thin 72 year old lady has a 3 week history of postprandial abdominal pain that is centrally located.
She has episodic diarrhoea and occasionally has passed blood PR. She has a history of ischaemic
heart disease and marked renal impairment from ACE inhibitor usage.

You answered Meseneric angiography via brachial artery

The correct answer is Duplex ultrasound

She is likely to have mesenteric vascular disease. Proximal SMA disease would be the most serious
variant. Ideally a CT angiogram would be the best test but with her impaired renal function and low
BMI, make a duplex of the SMA is a reasonable first line investigation. Gut peristalsis may impair
acquisition of magnetic resonance images.

59. A 78 year old man develops sudden onset abdominal pain and almost immediately afterwards
passes a large amount of diarrhoea.

You answered Abdominal CT with venous phase contrast


The correct answer is CT angiogram

Sudden onset of abdominal pain followed by forceful evacuation are the classical presenting
features of acute mesenteric infarction. This is best investigated by CT angiography, which has a
sensitivity of 95% for the diagnosis.

60. A 28 year old female has suffered from abdominal pain for the past 2 weeks since she was started
on the contraceptive pill. The pain has increased significantly over the post 10 hours and has been
associated with vomiting.

CT angiogram

Mesenteric venous thrombosis is the likely underlying cause and an angiogram is the sensible step
as it will also facilitate the identification of areas of infarcted bowel , similar to that which may
occur in the leg when massive DVT is present.

Vascular investigations

Venous disease

Venous Doppler
The simplest investigation for assessment of venous junctional incompetence is a Doppler assessment. This
involves the patient standing and manual compression of the limb distal to the junction of interest. Flow
should normally occur in one direction only. Where junctional incompetence is present reverse flow will occur
and is relatively easy to identify.

Venograms and duplex scans


Structural venous information is historically obtained using a venogram. This is an invasive test and rarely
required in modern clinical practice. The most helpful test is a venous duplex scan which will provide
information relating to flow and vessel characteristics. Duplex is also useful in providing vein maps for bypass
surgery.

Arterial disease

Ankle-brachial pressure
The ankle brachial pressure index measurement is an important investigation as it will allow classification of
the severity of the flow compromise present. False readings may occur in those with calcified vessels such as
diabetics and results in such settings should be interpreted with caution. When auscultating the vessel note
should be made of the character of the signal. Monophasic signals are associated with a proximal stenosis and
reduction in flow. Triphasic signals provide reassurance of a healthy vessel.

Arterial Duplex
As with the vein the duplex scan can provide a substantial amount of information about arterial patency and
flow patterns. In skilled hands they can provide insight as to the state of proximal vessels that are anatomically
inaccessible to duplex (e.g. Iliacs). Through assessment of distal flow patterns. It is an operator dependent test.
Conventional angiogram
Vessel puncture and catheter angiography is the gold standard method of assessing arteries. High quality
information can usually be obtained. Limitations of the technique include the risk of contrast toxicity and risks
of vessel damage. Severely calcified vessels may be difficult to puncture and in this situation a remote access
site (e.g. brachial) may be used. This technique is particularly useful in providing a distal arterial roadmap
prior to femoro-distal bypass.

CT angiography
These tests provide a considerable amount of structural and flow information. They require contrast and thus
carry the risks associated with this. They are particularly useful in the setting of GI bleeding as they are
rapidly available and can be performed by a non vascular radiologist. However, they lack the facility for
endovascular intervention. In general they do not provide high enough resolution for distal arterial surgery.

Magnetic resonance angiography


This has the advantage of being non-invasive and not using nephrotoxic contrast. Movement artifact remains a
problem in some sites and distal arterial resolution is imperfect.

Question 61 of 71
A 52 year old male attends the stroke unit with dizziness and vertigo while playing tennis. He is known to
have hypertension and a previous myocardial infarct. He now complains of right arm pain. What is the most
likely diagnosis?

A. Posterior circulation infarct

B. Vertebrobasilar aneurysm

C. Dissection of thoracic aorta

D. Subclavian steal syndrome

E. Left middle cerebral artery infarct

Subclavian steal syndrome characteristically presents with posterior circulation symptoms, such as dizziness
and vertigo, during exertion of an arm. There is subclavian artery steno-occlusive disease proximal to the
origin of the vertebral artery and is associated with flow reversal in the vertebral artery. Management involves
percutaneous transluminal angioplasty or a stent.

Subclavian artery

Path

• The left subclavian comes directly off the arch of aorta


• The right subclavian arises from the brachiocephalic artery (trunk) when it bifurcates into the
subclavian and the right common carotid artery.
• From its origin, the subclavian artery travels laterally, passing between anterior and middle scalene
muscles, deep to scalenus anterior and anterior to scalenus medius. As the subclavian artery crosses
the lateral border of the first rib, it becomes the axillary artery. At this point it is superficial and within
the subclavian triangle.
Image sourced from Wikipedia

Branches

• Vertebral artery
• Internal thoracic artery
• Thyrocervical trunk
• Costocervical trunk
• Dorsal scapular artery

Question 62 of 71
A 74 year old lady has a long standing venous leg ulcer overlying her
medial malleolus. Which of the following statements relating to the
management of this condition is false?

A. Pentoxifylline may speed ulcer healing

B. Treatment with daily low dose flucloxacillin may speed


ulcer healing

C. Multilayer bandages may provide compression equivalent


to 40mmHg

D. Large ulcers may be considered for skin grafting

E. They should not be treated with compression stockings if


the ankle / brachial pressure index is 0.6

Routine use of antibiotics is not advised as this may predispose to


resistant organisms. Pentoxifylline was subjected to a Cochrane
review in 2007 and shown to improve healing rates.

Lower leg ulcers


Venous leg ulcers

• Most due to venous hypertension, secondary to chronic venous


insufficiency (other causes include calf pump dysfunction or
neuromuscular disorders)
• Ulcers form due to capillary fibrin cuff or leucocyte
sequestration
• Features of venous insufficiency include oedema, brown
pigmentation, lipodermatosclerosis, eczema
• Location above the ankle, painless
• Deep venous insufficiency is related to previous DVT and
superficial venous insufficiency is associated with varicose
veins
• Doppler ultrasound looks for presence of reflux and duplex
ultrasound looks at the anatomy/ flow of the vein
• Management: 4 layer compression banding after exclusion of
arterial disease or surgery
• If fail to heal after 12 weeks or >10cm2 skin grafting may be
needed

Marjolin's ulcer

Image sourced from Wikipedia

• Squamous cell carcinoma


• Occurring at sites of chronic inflammation e.g; burns,
osteomyelitis after 10-20 years
• Mainly occur on the lower limb

Arterial ulcers
• Occur on the toes and heel
• Painful
• There may be areas of gangrene
• Cold with no palpable pulses
• Low ABPI measurements

Neuropathic ulcers

• Commonly over plantar surface of metatarsal head and plantar


surface of hallux
• The plantar neuropathic ulcer is the condition that most
commonly leads to amputation in diabetic patients
• Due to pressure
• Management includes cushioned shoes to reduce callous
formation

Pyoderma gangrenosum

Image sourced from Wikipedia

• Associated with inflammatory bowel disease/RA


• Can occur at stoma sites
• Erythematous nodules or pustules which ulcerate

1/3 Question 63-65 of 71


Theme: Investigation of vascular disease

A. Angiography
B. Arterial duplex scan
C. Arch aortogram
D. CXR
E. CT scan
F. Venous duplex scan

Please select the most appropriate investigation for the scenario given. Each option may be used once, more
than once or not at all.

63. A 22 year old professional tennis player attends A&E with a swollen painful right arm. His fingers
are dusky.

You answered Arterial duplex scan

The correct answer is Venous duplex scan

This patient has an axillary vein thrombosis. It classically presents with pain and swelling of an
effort induced limb. Duplex scan is needed to exclude a thombus.

64. A 65 year old man presents with expressive dysphasia and left sided weakness over 4 hours. His
symptoms have now completely resolved.

You answered Angiography

The correct answer is Arterial duplex scan

This patient has had a transient ischaemic attack. He urgently needs carotid duplex scanning to
assess if he needs a carotid endarterectomy.

65. A 65 year old man presents, for the first time, with pain at the back of his calves when he mobilises
10 metres. He is known to have hypertension.

Arterial duplex scan

An arterial duplex should be performed first, before progression to an angiography.

3 / 3 Question 66-68 of 71
Theme: Ankle Brachial pressure index measurements

A. >1
B. 0.6- 0.8
C. 1
D. 0.4-0.6
E. <0.4

For the scenarios described below, please select the most likely ankle
brachial pressure index measurement. Each option may be used once, more
than once or not at all.
66. An 83 year old male with rest pain.

<0.4

Theme from April 2012 Exam


Rest pain is typically associated with low ABPI values.

67. A 45 year old man who develops calf pain after walking 600
yards. It resolves during periods of rest.

0.6- 0.8

Since this is a long claudication distance it may be that only a


minor lesion is present. Whilst resting ABPI may be normal, they
are usually abnormal following exercise.

68. A 43 year old lady with long standing diabetes who complains of
calf pain. It is worse at night and during minor exercise.

>1

Diabetes may be complicated by vessel calcification and


neuropathic pain. Therefore individuals may present with pain
which is atypical for claudation both in terms of its tempo of onset
and location.

Ankle-Brachial pressure index

• Measurement of ankle- brachial pressure index (ABPI) is a


commonly performed vascular investigation.
• Calculated by dividing lower limb pressure by the highest upper
limb pressure.

Results of ABPI
1.2 or greater Usually due to vessel calcification
1.0- 1.2 Normal
0.8-1.0 Minor stenotic lesion
Initiate risk factor management
0.50-0.8 Moderate stenotic lesion
Consider duplex
Risk factor management
If mixed ulcers present then avoid tight compression
bandages
0.5- 0.3 Likely significant stenosis
Duplex scanning to delineate lesions needed
Compression bandaging contra indicated
Less than 0.3 Indicative of critical ischaemia
Urgent detailed imaging required

3/3 Question 69-71 of 71


Theme: Causes of ulceration

A. Marjolin's ulcer
B. Neuropathic ulcer
C. Arterial ulcer
D. Deep venous dysfunction
E. Superficial venous dysfunction
F. Rheumatoid arthritis
G. Pyoderma gangrenosum
H. Pressure ulcer

Please select the most likely cause of ulceration for the scenario given. Each option may be used once, more
than once or not at all.

69. A 62 year old diabetic man presents with long standing plantar ulcer he has clinical evidence of a
charcot foot.

Neuropathic ulcer

Plantar ulcers in association with peripheral neuropathy are often neuropathic. They classically
occur at pressure points.

70. A 66 year old female has long standing mixed arteriovenous ulcers of the lower leg. Over the past 6
months one of the ulcers has become much worse and despite a number of different topical
therapies is increasing in size.

Marjolin's ulcer

Marjolin's ulcer is a squamous cell carcinoma occurring at sites of chronic inflammation or previous
injury.

71. A 28 year old man undergoes a ileocaecal resection and end ileostomy for Crohn's disease. One year
later he presents with a deep painful ulcer at his stoma site.

Pyoderma gangrenosum

Pyoderma gangrenosum is associated with inflammatory bowel disease (this patient had a stoma for
crohns!). It is commonly found on lower limbs and described as being painful, the size of an insect
bite and growing. It looks like a margarita pizza (with a red base and yellow topping!) Treatment
involves steroids.

You might also like